Top Banner
TEST - 15 (C) Insights Active Learning. | All rights reserved. www.insightsias.com 1 1 Which of the following correctly point out the geo-strategic significance of the Chumbi valley? It is at the intersection of Sikkim, Bhutan and Tibet in the Himalayas. 1. Two main passes including the Nathu la pass between India and China open up here. 2. Which of the above is/are correct? 1 only A. 2 only B. Both 1 and 2 C. None D. User Answer : Correct Answer : C Answer Justification : Justification:Statement 1: It is in Tibet at the intersection of India (Sikkim), Bhutan and China (which claims Tibet) in the Himalayas. So, it is crucial to settling the territorial disputes between India and China in Tibet. Statement 2: The Nathu La Pass and Jelep La Pass open up here. Nathu La is one of the two open trading border posts between China and India which was closed following the 1962 war and re-opened in 2006. Q Source:Physical features of India: Chapter 1: India Yearbook 2016 2 Consider the following statements. Assertion (A): A state legislature cannot impose any taxes on the sale or purchase of 1. goods on its own. Reason (R): A state legislature needs the approval of the President for imposing any 2. tax. In the context of the above, which of these is correct? A is correct, and R is an appropriate explanation of A. A. A is correct, but R is not an appropriate explanation of A. B. A is incorrect, but R is correct. C. Both A and R are incorrect. D. User Answer : Correct Answer : D Answer Justification : Justification: A state legislature can impose taxes on professions, trades, callings and employments, sale or purchase of goods (other than newspapers) etc. However, a tax imposed on the sale or purchase of goods declared by Parliament to INSIGHTS IAS
66

Test 15 Solutions

Jul 07, 2016

Download

Documents

Ankit Gupta

ccse
Welcome message from author
This document is posted to help you gain knowledge. Please leave a comment to let me know what you think about it! Share it to your friends and learn new things together.
Transcript
Page 1: Test 15 Solutions

TEST - 15

(C) Insights Active Learning. | All rights reserved. www.insightsias.com 1

1 Which of the following correctly point out the geo-strategic significance of the Chumbivalley?

It is at the intersection of Sikkim, Bhutan and Tibet in the Himalayas.1.Two main passes including the Nathu la pass between India and China open up here.2.Which of the above is/are correct?

1 onlyA.2 onlyB.Both 1 and 2C.NoneD.

User Answer :Correct Answer : CAnswer Justification :

Justification:Statement 1: It is in Tibet at the intersection of India (Sikkim), Bhutanand China (which claims Tibet) in the Himalayas. So, it is crucial to settling theterritorial disputes between India and China in Tibet.

Statement 2: The Nathu La Pass and Jelep La Pass open up here. Nathu La is one ofthe two open trading border posts between China and India which was closedfollowing the 1962 war and re-opened in 2006.

Q Source:Physical features of India: Chapter 1: India Yearbook 2016

2 Consider the following statements.Assertion (A): A state legislature cannot impose any taxes on the sale or purchase of1.goods on its own.Reason (R): A state legislature needs the approval of the President for imposing any2.tax.In the context of the above, which of these is correct?

A is correct, and R is an appropriate explanation of A.A.A is correct, but R is not an appropriate explanation of A.B.A is incorrect, but R is correct.C.Both A and R are incorrect.D.

User Answer :Correct Answer : DAnswer Justification :

Justification: A state legislature can impose taxes on professions, trades, callingsand employments, sale or purchase of goods (other than newspapers) etc.

However, a tax imposed on the sale or purchase of goods declared by Parliament to

INSIG

HTS IAS

Page 2: Test 15 Solutions

TEST - 15

(C) Insights Active Learning. | All rights reserved. www.insightsias.com 2

be of special importance in inter-state trade and commerce is subject to therestrictions and conditions specified by the Parliament.

Presidential assent is not generally required for introduction of taxes by a state

Q Source: Chapter 14: Indian Polity: M Laxmikanth

3 They are a series of parallel ridges arcing through Assam, Nagaland, Mizoram, Burmaand are submerged in the Bay of Bengal for a long stretch emerging again in the form ofthe Andaman and Nicobar Islands. The mountains are

Namcha Barwa RangesA.Trans-Alay RangeB.Dhaulagiri MountainsC.Arakan MountainsD.

User Answer :Correct Answer : DAnswer Justification :

Learning: The Arakan Mountains (also called Rakhine ranges) and the parallel arcsto the west and east were formed by compression as the Indian Plate collided withthe Eurasian Plate approximately along the boundary between India and Nepal.

The Arakan Mountains divide the Rakhine coast from the rest of Burma, and thushave acted as a barrier between the peoples of central Burma and those of the Indiansubcontinent.

The Arakan Mountains act as a barrier to the south-western monsoon rains and thusshield the central Myanmar area, making their western slopes extraordinarily wetduring the monsoon.

Q Source: Physical features of India: Chapter 1: India Yearbook 2016

4 Consider the following about Madan Mohan Malaviya.He took a vow not to join any legislative council.1.He was the founder of Ganga Mahasabha.2.He opposed the separate electorates for Muslims under the Lucknow Pact of 1916.3.He was a President of the Indian National Congress (INC).4.Select the correct answer using the codes below.

2, 3 and 4 onlyA.4 onlyB.

INSIG

HTS IAS

Page 3: Test 15 Solutions

TEST - 15

(C) Insights Active Learning. | All rights reserved. www.insightsias.com 3

1 and 3 onlyC.1, 2, 3 and 4D.

User Answer :Correct Answer : AAnswer Justification :

Justification: Statement 1: He was a member of the Imperial Legislative Councilfrom 1912 and when in 1919 it was converted to the Central Legislative Assemblyhe remained its member till 1926.

Statement 2: He founded it at Haridwar in 1905. He was a member of the HinduMahasabha.

Statement 3: He was a moderate leader and opposed communal politics by theBritish. He was an important figure in the Non-cooperation movement. However, hewas opposed to the politics of appeasement by Congress and its participation in theKhilafat movement.

Statement 4: Malaviya was the President of the Indian National Congress on twooccasions (1909, 1918). He left Congress in 1934.

Q Source: Next to Cover page: India Yearbook 2016

5 Consider the following statements.Assertion (A): No river originating in Rajasthan meets the sea.1.Reason (R): A large part of Rajasthan is desert.2.In the context of the above, which of these is correct?

A is correct, and R is an appropriate explanation of AA.A is correct, but R is not an appropriate explanation of A.B.A is incorrect, but R is correct.C.Both A and R are correct independently.D.

User Answer :Correct Answer : CAnswer Justification :

Justification: Sabarmati River is one example that originates in Rajasthan(Udaipur) and meets the Arabian Sea.

Only a few rivers in Rajasthan do not drain into the sea. They drain into salt lakesand get lost in sand with no outlet to sea.

INSIG

HTS IAS

Page 4: Test 15 Solutions

TEST - 15

(C) Insights Active Learning. | All rights reserved. www.insightsias.com 4

Besides these, there are the Desert Rivers which flow for some distance and are lostin the desert. These are Luni, Machhu, Rupen, Saraswati, Banas, Ghaggar andothers.

Q Source:Physical features of India: Chapter 1: India Yearbook 2016

6 Which of the following have been introduced by foreigners in India?Potatoes1.Tomato2.Red Chilli3.Tea and Coffee4.Peanuts and Almond5.Select the correct answer using the codes below.

3, 4 and 5 onlyA.4 and 5 onlyB.1, 2, 3 and 4 onlyC.1, 2, 3, 4 and 5D.

User Answer :Correct Answer : CAnswer Justification :

Learning: Statement 1: When the Dutch came to India, they introduced the cultureof potatoes, and from them the British received various kinds of potatoes.

Statement 2: Originating in South America, it may have been introduced as late as1850 into India.

Statement 3: It arrived in India during 16th century replacing Pippali (long pepper).

Statement 4: Tea was first introduced into India by the British, in an attempt tobreak the Chinese monopoly on tea.

The first record of coffee growing in India is following the introduction of coffeebeans from Yemen by Baba Budan to the hills of Chikmagalur in 1670.

Q Source: Revision Previous Tests Syllabus: 11th Geography NCERT: India,People and Economy

7 Consider the following about the protocol followed while hoisting the National Flag.The Flag should never touch the ground or water.1.

INSIG

HTS IAS

Page 5: Test 15 Solutions

TEST - 15

(C) Insights Active Learning. | All rights reserved. www.insightsias.com 5

Indian citizens can fly the flag even at night.2.The Flag should not hold any objects other than flower petals before unfurling.3.No lettering should be inscribed on the flag.4.The flag cannot be used in uniforms.5.Select the correct answer using the codes below.

2 and 4 onlyA.3, 4 and 5 onlyB.1, 2 and 3 onlyC.1, 2, 3 and 4 onlyD.

User Answer :Correct Answer : DAnswer Justification :

Learning: The original flag code also forbade use of the flag on uniforms, costumesand other clothing.

In 2005, the Government of India amended the code to allow some forms of usage.The amended code forbids usage in clothing below the waist and on undergarments,and forbids embroidering onto pillowcases, handkerchiefs or other dress material.

Display and usage of the flag is governed by the Flag Code of India, 2002(successor to the Flag Code - India, the original flag code); the Emblems and Names(Prevention of Improper Use) Act, 1950; and the Prevention of Insults to NationalHonour Act, 1971.

Q Source: National Flag: Chapter 2: India Yearbook 2016

8 World Consumer Rights Day is observed on 15th March to commemorate the historicaddress given by the then US President John F Kennedy to the US congress in 1962. InIndia, 24th December is observed as National Consumer Day. On this day

Consumer Protection Act, 1986 had received the assent of the President.A.Dandi March started which was the first ever consumer rights movement inB.India.First consumer dispute was resolved by the Supreme Court.C.The first ever consumer case came to light in independent India.D.

User Answer :Correct Answer : AAnswer Justification :

Learning: 2016 Theme of the World Consumer Rights Day: "Antibiotics off themenu" - The theme is a campaign to stop the sale of meat raised with the use of

INSIG

HTS IAS

Page 6: Test 15 Solutions

TEST - 15

(C) Insights Active Learning. | All rights reserved. www.insightsias.com 6

antibiotics important to human medicine.

It seeks to spread awareness about antibiotic resistance driven by overuse ofantibiotics in agriculture to promote faster growth and to prevent diseases ratherthan treat diseases.

Q Source: http://pib.nic.in/newsite/PrintRelease.aspx?relid=133634

9 In the Indian National Saka Calendar, the months in the first half of the year all have 31days to take into account

Change of seasonsA.Slower movement of the sun across the eclipticB.Old traditions and festivalsC.Orbital velocity of the MoonD.

User Answer :Correct Answer : BAnswer Justification :

Justification: The ecliptic is the apparent path of the Sun on the celestial sphere. Itis coplanar (same plane) with the orbit of Earth around the Sun.

The Sun seems to move against the background stars as seen from the orbitingEarth. The ecliptic is the path the Sun appears to trace through the stars.

Because Earth takes one year to orbit the Sun, the apparent position of the Sun alsotakes the same length of time to make a complete circuit of the ecliptic.

But, the actual speed with which Earth orbits the Sun varies slightly during the year,so the speed with which the Sun seems to move along the ecliptic also varies. This istaken into account by the Saka Calendar and the first six months are adjusted for theapparent slow movement of the Sun in the ecliptic.

Q Source: National Calendar: Chapter 2: India Yearbook 2016

10 The provision that Speaker does not vacate her office until immediately before the firstmeeting of the House after dissolution is found in the

Parliamentary rules and procedureA.Constitution of IndiaB.Representation of People of India Act, 1950C.Conventions of the Indian ParliamentD.

INSIG

HTS IAS

Page 7: Test 15 Solutions

TEST - 15

(C) Insights Active Learning. | All rights reserved. www.insightsias.com 7

User Answer :Correct Answer : BAnswer Justification :

Justification: Article 94 of the constitution deals with the vacation/ resignation/removal from the offices of the Speaker and Deputy Speaker.

It provides that whenever the House of the People is dissolved, the Speaker shall notvacate his office until immediately before the first meeting of the House of thePeople after the dissolution.

Q Source: Table 3.1: Chapter 3: India Yearbook 2016

11 Which of the following departments do NOT belong to the Union Ministry of HomeAffairs?

Department of Administrative Reforms1.Department of Defence Production2.Department of States3.Department of Home4.Department of Ex-servicemen Welfare5.Department of Official Language6.Select the correct answer using the codes below.

1, 2, 4 and 5 onlyA.1, 2 and 5 onlyB.2, 3, 4 and 5 onlyC.1 and 6 onlyD.

User Answer :Correct Answer : BAnswer Justification :

Justification: 2 and 5 are in Ministry of Defence. 1 is in the Ministry of Personnel,Public Grievances and Pensions.

Departments under Ministry of Home Affairs are:

Department of Internal Security (Aantarik Suraksha Vibhag)Department of States (Rajya Vibhag)Department of Official Language (Raj Bhasha Vibhag)Department of Home (Grih Vibhag)Department of Jammu and Kashmir Affairs (Jammu tatha Kashmir Vibhag)Department of Border Management (Seema Prabandhan Vibhag)

INSIG

HTS IAS

Page 8: Test 15 Solutions

TEST - 15

(C) Insights Active Learning. | All rights reserved. www.insightsias.com 8

Q Source: Chapter 3: India Yearbook 2016

12 As per the Aadhaar (Target Delivery of Financial and Other Subsidies, Benefits andServices) Bill, 2016 passed by Lok Sabha

Aadhaar can now be used as a proof of citizenship or domicile in India.1.Biometric information collected during the Aadhaar enrolment can now be freely2.used by the Police for investigating crimes.Which of the above is/are correct?

1 onlyA.2 onlyB.Both 1 and 2C.NoneD.

User Answer :Correct Answer : DAnswer Justification :

Justification: Statement 1: Use of Aadhaar number will be for: (i) Verifying theidentity of a person receiving a subsidy or a service. (ii) as a proof of identity of theAadhaar number holder for any purpose asked by any public or private entity. (iii) Itcannot be a proof of citizenship or domicile.

Statement 2: Biometric information will be used only for Aadhaar enrolment andauthentication and for no other purpose. Such information will not be shared withanyone, nor displayed publicly, except for purposes specified by regulations, i.e.national security and court orders.

Q Source:http://www.thehindu.com/news/national/lok-sabha-passes-aadhaar-bill/article8341908.ece

13 The Krishonnati Yojana is a new umbrella scheme submerging which of the followingexisting scheme(s)?

Soil Health Card scheme1.Paramparagat Krishi Vikas Yojana2.National Food Security Mission3.Select the correct answer using the codes below.

1 and 2 onlyA.3 onlyB.2 and 3 onlyC.1, 2 and 3D.

INSIG

HTS IAS

Page 9: Test 15 Solutions

TEST - 15

(C) Insights Active Learning. | All rights reserved. www.insightsias.com 9

User Answer :Correct Answer : BAnswer Justification :

Justification: National Food Security Mission was launched in 2007-08 to increasethe production of rice, wheat and pulses

Paramparagat Krishi Vikas Yojana is a cluster based programme to encourage thefarmers for promoting organic farming.

Under the Soil Health Card Scheme, Soil Health Card (SHC) will be provided to allfarmers in the country at an interval of three years so as to enable them to applyappropriate recommended dosages of nutrients for crop production and improvingsoil health and its fertility.

Q Source: Chapter 4: India Yearbook 2016

14 Which of the following statements is INCORRECT about Indicative planning?Mixed economies can follow indicative planning.1.The state does not play any role in indicative planning.2.It signals liberalization of market forces.3.No state owned companies are allowed to operate in the economy.4.Select the correct answer using the codes below.

2, 3 and 4 onlyA.1 and 2 onlyB.2 and 4 onlyC.4 onlyD.

User Answer :Correct Answer : CAnswer Justification :

Justification: Statement 1 and 2: Capitalist economies do not involve the state inthe planning process. And, in communist economies the planning is imperative notindicative in nature. So, mixed economies are fit for indicative planning where aplan making body (from the State) sets broad numerical targets to be achieved withthe help of market forces.

Statement 3: Unlike a centrally planned economy indicative planning works throughthe market (price system) rather than replaces it. So, it is a move towardsliberalization of market forces

INSIG

HTS IAS

Page 10: Test 15 Solutions

TEST - 15

(C) Insights Active Learning. | All rights reserved. www.insightsias.com 10

Statement 4: India followed indicative planning, but the PSUs still continue tooperate.

Q Source: Types of Planning: Chapter 4: Indian Economy - Ramesh Singh

15 Which of the following is/are member countries of BIMSTEC (Bay of Bengal Initiativeon Multi Sectoral Technical and Economic Cooperation)?

Nepal1.Bhutan2.Myanmar3.Thailand4.Sri Lanka5.China6.Select the correct answer using the codes below

All except 3 and 4A.All except 1 and 2B.All except 5 and 6C.All except 6D.

User Answer :Correct Answer : DAnswer Justification :

Learning: It is the sub-regional group of seven countries in South Asia and SouthEast Asia.

They are India, Nepal, Bangladesh, Bhutan, Sri Lanka (from South Asia) andMyanmar, Thailand (from South East Asia).

It was established in 1997. Headquarters is Dhaka, Bangladesh.

It also covers cooperation in commerce, technology, investment, agriculture,tourism, human resource development, fisheries, transport and communication,textiles, leather etc.

Q Source: Revision Previous Tests Syllabus: International and Regionalorganizations

16 "The Economic History of India" written in the 20th century which examined in minutedetail the entire economic record of colonial rule since 1757 was written by

Mahadev Govind RanadeA.

INSIG

HTS IAS

Page 11: Test 15 Solutions

TEST - 15

(C) Insights Active Learning. | All rights reserved. www.insightsias.com 11

G. Subramaniya lyerB.G.K. GokhaleC.Romesh Chandra DuttD.

User Answer :Correct Answer : DAnswer Justification :

Learning: The book explores the drain of wealth from India. It shows how specificpolicies of British in the sphere of railways, cotton, plantation agriculture,handicrafts led to the ruin of Indians and India's economy.

Romesh Chandra Dutt was active in moderate nationalist politics and was an activeCongressman in that party's initial phase.

He was also a president of the Indian National Congress in 1899.

Q Source: Page 68: Chapter 7: Bipin Chandra: India's struggle for Independence

17 Consider the following about 'One MP - One Idea Scheme'.It was initiated by the Ministry of Parliamentary affairs.1.The District administration submits innovative ideas to MPs who then forward it to2.the Union Cabinet for ratification and implementation.Ideas must be funded by the concerned MPs through MPLADS funds.3.Select the correct answer using the codes below.

1 and 2 onlyA.2 onlyB.2 and 3 onlyC.None of the aboveD.

User Answer :Correct Answer : DAnswer Justification :

Justification: Statement 1: It was announced by the Ministry of Statistics andProgramme Implementation under the Member of Parliament Local AreaDevelopment Scheme (MPLADS).

Statement 2: Based on the innovative ideas received from the local people regardingdevelopmental projects, a 'One MP - One Idea' Competition can be held in each LokSabha constituency annually to select the three best innovations for cash awards andcertificate of appreciation for next five best innovations.

INSIG

HTS IAS

Page 12: Test 15 Solutions

TEST - 15

(C) Insights Active Learning. | All rights reserved. www.insightsias.com 12

A committee headed by DM evaluates these ideas.

Statement 3: MPs award Certificate of Honour along with cash award of Rs. 2.5lakhs, Rs. 1.5 lakhs and Rs. 1 lakh to the 1st , 2nd and 3rd best innovative solutionsrespectively through their MPLAD funds.

Lead banks and financial institutions fund local innovations.

An overview presentation on the 'One MP - One Idea' initiative can be viewed here

http://innovationcouncilarchive.nic.in/index.php?option=com_content&view=article&id=343&Itemid=121

Q Source: MPLADS: Chapter 5: Indian Economy - Ramesh Singh

18 Consider the following about Tattwabodini Sabha.It was a splinter group of the Brahmo Samaj.1.It was founded by Debendranath Tagore.2.Its main objective was to promote a rational and humanist form of Hinduism based3.on the Vedanta.Select the correct answer using the codes below.

1 and 2 onlyA.3 onlyB.2 and 3 onlyC.1, 2 and 3D.

User Answer :Correct Answer : DAnswer Justification :

Learning: As per some commentators, "Their view, at least in the early years, wasthat the world is created by God, and all things within it are pathways to knowledgeof Brahman, the Ultimate Self, and the ultimate goal. Similarly, they saw thatmaterial wealth, if made and possessed with the correct intention - that of helpingsociety and others - was in fact not only ethically sound, but an utter necessity forharmonious society."

In 1859, the Sabha was dissolved back into the Brahmo Samaj by DebendranathTagore.

Q Source: Page 65: Chapter 6: Bipin Chandra: India's struggle for Independence

INSIG

HTS IAS

Page 13: Test 15 Solutions

TEST - 15

(C) Insights Active Learning. | All rights reserved. www.insightsias.com 13

19 Which of the following measures may help tackle inflation?Import of goods which are in short-supply1.Technological innovations that reduce the production cost of goods2.Improving Supply-side infrastructure3.Tight monetary policy4.Cracking down on illegal hoarding of sensitive items5.Select the correct answer using the codes below.

1, 3 and 4 onlyA.1, 2 and 5 onlyB.3, 4, 5 onlyC.1, 2, 3, 4 and 5D.

User Answer :Correct Answer : DAnswer Justification :

Justification: Statement 1, 3 and 5: It has happened in India in the case of onion andpulses and meeting the buffer stock norm of wheat. As a long-term measure,governments go on to increase the production to matching the level of demand.Storage, transportation, distribution, hoarding are the other aspects of pricemanagement of this category.

Statement 2: For e.g. Cost of energy (electricity) has effectively come down due tointroduction of LED bulbs. Any possible inflation in energy prices are also takencare by the low power consumption and electricity bills due to LEDs.

Statement 4: It is basically intended to cut down the money supply in the economyby siphoning out the extra money (as RBI increases the Cash Reserve Ratio of bankin India)9 from the economy and by making money costlier (as RBI increases theBank Rate or Repo Rate in India).

This is a short-term measure. In the long-run, the best way is to increase productionwith the help of the best production practices.

Q Source: Chapter 7: Indian Economy - Ramesh Singh

20 Washington Consensus refers toA set of economic reform prescriptions for developing countriesA.A strategy to tackle organized terrorism in the Middle-EastB.A set of prescriptions for ecologically sustainable mining in erstwhileC.colonial African countriesA plan by UNFCCC to reverse climate change and save low lying islandD.

INSIG

HTS IAS

Page 14: Test 15 Solutions

TEST - 15

(C) Insights Active Learning. | All rights reserved. www.insightsias.com 14

nationsUser Answer :Correct Answer : AAnswer Justification :

Learning: It is considered to constitute the "standard" reform package promoted forcrisis-wracked developing countries by Washington, D.C.-based institutions such asthe International Monetary Fund (IMF), World Bank, and the US TreasuryDepartment.

It basically refers to a more general orientation towards a strongly market-basedapproach such as macroeconomic stabilization, economic opening with respect toboth trade and investment, and the expansion of market forces within the domesticeconomy.

Q Source: Chapter 6: Indian Economy - Ramesh Singh

21 Consider the following statements.He started the first Tamil newspaper Swadeshmitram in 1882.1.He founded 'The Hindu' newspaper and was also its editor till 1898.2.The above refer to?

G. Subrahmaniya IyerA.Vaidyanatha IyerB.Alluri Sita Rama RajuC.Chinnaswami Subramanya BharathiyarD.

User Answer :Correct Answer : AAnswer Justification :

Learning: He was proprietor, editor and managing director of The Hindu

Subramania Iyer actively participated in the Indian Independence movement.

Subramania Iyer campaigned vehemently for reforms in Hindu society. Hesupported widow remarriage and desired to abolish untouchability and childmarriages.

In 1898, Subramania Iyer relinquished his claims over 'The Hindu' and concentratedhis energies on Swadesamitran, the Tamil language newspaper which he had startedin 1882.

INSIG

HTS IAS

Page 15: Test 15 Solutions

TEST - 15

(C) Insights Active Learning. | All rights reserved. www.insightsias.com 15

Q Source: Page 79: Chapter 8: Bipin Chandra: India's struggle for Independence

22 Consumer Price Index (CPI) was published by the Government across variouscategories like Industrial workers. How were these various categories useful from aneconomic perspective?

They formed the basis for Pay Commission's recommendations.1.They were used to deflate GDP of certain sectors to obtain real GDP of that sector.2.They were used to revise minimum wages for agricultural labourers.3.They showed the price impact on the most vulnerable segments of India.4.Select the correct answer using the codes below.

1, 3 and 4 onlyA.2 and 3 onlyB.4 onlyC.1, 2, 3 and 4D.

User Answer :Correct Answer : DAnswer Justification :

Justification: It is the Old CPI index. New Index is published across Rural, Urbanand Combined categories.

Statement 1: The Consumer Price Index for the industrial workers (CPI-IW)specifies the government employees (other than banks' and embassies' personnel).The wages/salaries of the central government employees are revised on the basis ofthe changes occurring in this index, the dearness allowance (DA) is announced twicea year.

When the Pay Commissions recommend pay revisions, the base is the CPI (IW).

Statement 2: The Consumer Price Index for the Urban Non-Manual Emplyees (CPI-UNME) is basically used for determining dearness allowances (DAs) of employeesof some foreign companies operating in India (i.e. airlines, communications,banking, insurance, embassies, and other financial services).

It is also used under the Income Tax Act to determine capital gains and by the CSO(Central Statistical Organisation) for deflating selected service sector's contributionto the GDP at factor cost and current prices to calculate the corresponding figure atconstant prices.

Statement 3 and 4: The Consumer Price Index for Agricultural Labourers (CPI-AL)is used for revising minimum wages for agricultural labourers in different states.

INSIG

HTS IAS

Page 16: Test 15 Solutions

TEST - 15

(C) Insights Active Learning. | All rights reserved. www.insightsias.com 16

The governments at the centre and states remain vigilant regarding the changes inthis index as it shows the price impact on the most vulnerable segment of thesociety, this segment spends almost 75 per cent of its total income on the purchaseof food articles.

Q Source: Chapter 7: Indian Economy - Ramesh Singh

23 The President is bound by the aid and advice tendered by the Council of Ministers. Thisprovision is

Mandated by Representation of People of India ActA.Followed as a convention of Parliamentary form of GovernmentB.An executive precedent followed since IndependenceC.Mandated by a Constitutional amendmentD.

User Answer :Correct Answer : DAnswer Justification :

Learning: Article 74 provides for a council of ministers headed by the PrimeMinister to aid and advise the President in the exercise of his functions. The adviceso tendered is binding on the President

Unamended constitution had some scope of Presidential discretion where he was notexplicitly bound by the aid and advice of the council of Ministers.

Q Source: Chapter 12: Indian Polity: M Laxmikanth

24 Consider the following about President's rule that is imposed under Article 356 of theConstitution.

It cannot be imposed without the written recommendation of the Governor of the1.concerned state.Every proclamation of President's rule must be approved by both the houses of2.Parliament within a stipulated time.Which of the above is/are correct?

1 onlyA.2 onlyB.Both 1 and 2C.NoneD.

User Answer :Correct Answer : BAnswer Justification :

INSIG

HTS IAS

Page 17: Test 15 Solutions

TEST - 15

(C) Insights Active Learning. | All rights reserved. www.insightsias.com 17

Learning: Statement 1: Article 356 empowers the President to issue a proclamation,if he is satisfied that a situation has arisen in which the government of a state cannotbe carried on in accordance with the provisions of the Constitution.

Notably, the president can act either on a report of the governor of the state orotherwise too (ie, even without the governor's report).

Statement 2: A proclamation imposing President's Rule must be approved by boththe Houses of Parliament within two months from the date of its issue. If approvedby both the Houses of Parliament, the President's Rule continues for six months

Q Source: Chapter 16: Indian Polity: M Laxmikanth

25 Some inflation is necessary for boosting investments in the short-run. This happensbecause

Higher inflation indicates higher demand in the economy1.Higher inflation lowers the cost of credit for entrepreneurs2.Which of the above is/are correct?

1 onlyA.2 onlyB.Both 1 and 2C.NoneD.

User Answer :Correct Answer : CAnswer Justification :

Justification: Statement 1: Higher inflation is because demand has outstrippedsupply. When this happens, entrepreneurs are motivated to expand production tofulfil the excess demand. This boosts investment in the economy.

Statement 2: Inflation redistributes wealth from creditors to debtors i.e. lenderssuffer and borrowers benefit out of inflation. The concept has also been covered inan earlier test. So, entrepreneurs (who borrow money for investing) benefit frominflation in repaying their loans.

Q Source: Chapter 7: Indian Economy - Ramesh Singh

26 Which of the following cases in the Supreme Court of India is related to resolving theconflict between Fundamental Rights and Directive Principles?

Golaknath case1.

INSIG

HTS IAS

Page 18: Test 15 Solutions

TEST - 15

(C) Insights Active Learning. | All rights reserved. www.insightsias.com 18

Champakam Dorairajan case2.Minerva Mills case3.Bhikan Singh case4.Select the correct answer using the codes below.

1 and 3 onlyA.2, 3 and 4 onlyB.1 and 4 onlyC.1, 2 and 3 onlyD.

User Answer :Correct Answer : DAnswer Justification :

Learning: In the Champakam Dorairajan case, the Supreme Court ruled that in caseof any conflict between the Fundamental Rights and the Directive Principles, theformer would prevail.

In Golaknath case (1967) the Supreme Court ruled that the Parliament cannot takeaway or abridge any of the Fundamental Rights, which are 'sacrosanct' in nature.

In the Minerva Mills case (1980), the Supreme Court also held that 'the IndianConstitution is founded on the bedrock of the balance between the FundamentalRights and the Directive Principles.

Q Source: Chapter 8: Indian Polity: M Laxmikanth

27 The Arab League is a regional organization of Arab countries in and around NorthAfrica, the Horn of Africa and Arabia. Member countries of the Arab League are

Sudan1.Iraq2.Iran3.Jordan4.Libya5.Somalia6.Select the correct answer using the codes below

2, 4 and 5 onlyA.1, 4 and 6 onlyB.3, 5 and 6 onlyC.1, 2, 4, 5 and 6 onlyD.

User Answer :Correct Answer : D

INSIG

HTS IAS

Page 19: Test 15 Solutions

TEST - 15

(C) Insights Active Learning. | All rights reserved. www.insightsias.com 19

Answer Justification :

Learning: The Arab League occupies an area spanning around 14 million km² andcounts 22 members, and 4 observer states. The 22 members today include 3 of thelargest African countries (Sudan, Algeria and Libya) and the largest country in theMiddle East (Saudi Arabia).

Syria was suspended following the 2011 uprising, but its seat was later given toSyrian opposition.

It was founded in 1945. It aimed to be a regional organisation of Arab states with afocus to developing the economy, resolving disputes and coordinating political aims.

Q Source: Revision Previous Tests Syllabus: International and Regionalorganizations

28 Amrita Bazaar Patrika played a major role in the evolution and growth of Indianjournalism and and nurturing the Indian freedom struggle. Consider the following about it.

It is the oldest Indian-owned English daily.1.It was started by Sisir Ghosh and Moti Lal Ghosh.2.It was discontinued before the Quit India Movement.3.Select the correct answer using the codes below.

1 and 2 onlyA.2 and 3 onlyB.1 and 3 onlyC.1, 2 and 3D.

User Answer :Correct Answer : AAnswer Justification :

Learning: It was launched in Bengali and later continue in English.

It used to be a nationalist newspaper during the British rule, discontinued itspublication from 1986.

It was the first Indian-owned English daily to go into investigativejournalism.The Patrika had many brushes with Lord Curzon, the Viceroy of India at thetime of the Partition of Bengal (1905).It referred to him as 'Young and a little foppish, and without previous trainingbut invested with unlimited powers.'

INSIG

HTS IAS

Page 20: Test 15 Solutions

TEST - 15

(C) Insights Active Learning. | All rights reserved. www.insightsias.com 20

Because of such editorials, the Press Act of 1910 was passed and MotilalGhosh was also charged with sedition.

Q Source: Page 83: Chapter 8: Bipin Chandra: India's struggle for Independence

29 The Union Government has launched an online marketing platform Mahila E-HAAT tofacilitate women entrepreneurs to sell their products to buyers. It is a joint initiative of

Department of Electronics and Information Technology and All IndiaA.Women's Federation (AIWF)Union Women and Child Development Ministry and Rashtriya Mahila KoshB.(RMK)Commission on the Status of Women (CSW) and Union Women and ChildC.Development MinistryDepartment of Electronics and Information Technology and Self-EmployedD.Women's Association (SEWA)

User Answer :Correct Answer : BAnswer Justification :

Learning: It is an initiative mainly for women across the country and is part ofUnion Government's flagship 'Digital India' and 'Stand Up India' initiatives.

Its objective is to strengthen the financial inclusion of women entrepreneurs byproviding continued support to their creativity

The initiative is unique as for the first time government will help women tosell products online.Women can sell their products like cloths, bags, jewelleries through this webportal.It is open to all Indian women citizens above 18 years of age and women SelfHelp Groups (SHGs) desiring for marketing their legal products/services. Theentire business on the e-Haat platform can be easily handled through mobilephone.

Q Source: http://pib.nic.in/newsite/PrintRelease.aspx?relid=137415

30 Which of the following statements about Ishwarchandra Vidyasagar is correct?He was a key figure of the Bengal Renaissance.1.He took the initiative in proposing and pushing the Widow Remarriage Act 1856.2.Which of the above is/are correct?

INSIG

HTS IAS

Page 21: Test 15 Solutions

TEST - 15

(C) Insights Active Learning. | All rights reserved. www.insightsias.com 21

1 onlyA.2 onlyB.Both 1 and 2C.NoneD.

User Answer :Correct Answer : CAnswer Justification :

Learning: He also demonstrated that the system of polygamy without restrictionwas not sanctioned by the ancient Hindu Shastras.

He reconstructed the Bengali alphabet and reformed Bengali typography into analphabet of twelve vowels and forty consonants. He contributed significantly toBengali and Sanskrit literature.Vidyasagar's "Barna Porichoy" is still considered aclassic.

Q Source: Page 58: Chapter 6: Bipin Chandra: India's struggle for Independence

31 Consider the following.The economy heats up and a demand-supply lag is visible.1.There can be shortage of investible capital and savings2.A seller's market may be created.3.The above may be associated with which of the following phases of business cycle?

DepressionA.BoomB.RecessionC.StagflationD.

User Answer :Correct Answer : BAnswer Justification :

Justification: Statement 2: Due to a mismatch between supply and demand, andhigher consumption (and lower savings), there can be a shortage of investiblecapital.

Due to this the economy heats up and supply-side bottlenecks become clearlyvisible. So, statement 1 is also associated here.

Overheating of an economy occurs when its productive capacity is unable to keeppace with growing aggregate demand leading to inflation.

INSIG

HTS IAS

Page 22: Test 15 Solutions

TEST - 15

(C) Insights Active Learning. | All rights reserved. www.insightsias.com 22

Statement 3: A seller's market is an economic situation in which goods or shares arescarce and sellers can keep prices high. During boom as demand outstrips supply,the situation can often be seen across.

Q Source: Chapter 7 - Section B - Business Cycle: Indian Economy - RameshSingh

32 It has been said that the merits of the ASEAN Way might "be usefully applied to globalconflict management". What is called as the 'ASEAN Way'?

It reflects the spiritual life style of ASEAN members countries.A.Involving third party countries in bilateral negotiationsB.It is a form of interaction among ASEAN members that is informal,C.cooperative and respects cultural norms.Never taking recourse to the judicial route to resolve commercial conflictsD.between nations

User Answer :Correct Answer : CAnswer Justification :

Learning: It is a working process or style that is informal and personal.

Policymakers constantly utilize compromise, consensus, and consultation in theinformal decision-making process. Above all it prioritizes a consensus-based, non-conflictual way of addressing problems.

Quiet diplomacy allows ASEAN leaders to communicate without bringing thediscussions into the public view. Members avoid embarrassment that may lead tofurther conflict.

Q Source: Revision Previous Tests: 12th NCERT: Contemporary World Politics

33 Consider the following statements.The governor can reserve certain types of bills passed by the state legislature for the1.consideration of the President.Bills on certain matters enumerated in the State List can be introduced in the state2.legislature only with the previous sanction of the president.The President can direct the states to reserve money bills and other financial bills3.passed by the state legislature for his consideration during a financial emergency.Select the correct answer using the codes below.

1 and 2 onlyA.

INSIG

HTS IAS

Page 23: Test 15 Solutions

TEST - 15

(C) Insights Active Learning. | All rights reserved. www.insightsias.com 23

1 and 3 onlyB.2 and 3 onlyC.1, 2 and 3D.

User Answer :Correct Answer : DAnswer Justification :

Learning: Besides the Parliament's power to legislate directly on the state subjectsunder the exceptional situations, the Constitution empowers the Centre to exercisecontrol over the state's legislative matters in the above mentioned ways.

For example, the bills imposing restrictions on the freedom of trade and commercecan be introduced only after Presidential assent.

Q Source: Chapter 14: Indian Polity: M Laxmikanth

34 The Strait of Gibraltar is a narrow strait that connectsAtlantic Ocean to the Mediterranean SeaA.Red Sea to Caspian SeaB.Mediterranean Sea to Black SeaC.Red Sea to Atlantic OceanD.

User Answer :Correct Answer : AAnswer Justification :

Learning: It separates Gibraltar and Peninsular Spain in Europe from Morocco andCeuta (Spain) in Africa.

The Strait has been identified as an Important Bird Area by BirdLife Internationalbecause hundreds of thousands of seabirds use it every year to pass between theMediterranean and the Atlantic.

The direct linkage of Atlantic Ocean to Mediterranean Sea creates certain uniqueflow and wave patterns. You can read about them here.

https://en.wikipedia.org/wiki/Strait_of_Gibraltar#Special_flow_and_wave_patterns

Q Source: Revision Previous Tests (UPSC CSP) + World Geography questions +Important Biodiversity Regions

INSIG

HTS IAS

Page 24: Test 15 Solutions

TEST - 15

(C) Insights Active Learning. | All rights reserved. www.insightsias.com 24

35 Vehicles carrying inflammable material usually have metal ropes touching the groundduring motion. This is because

It helps to ignite engine spark easily in such heavily loaded vehiclesA.Velocity of the vehicle can be controlled this way.B.The metal rope conducts the charge produced by air-friction in the vehicleC.The vehicle needs to be insulated against lightning.D.

User Answer :Correct Answer : CAnswer Justification :

Justification: When a vehicle moves, the body also gets charged due to air-frictionalong with the tyres.

If vehicle carries inflammable material, the charge accumulated on the body maycause fire.

To avoid this, the accumulated charge is conducted to earth through the metallicropes.

Q Source: Revision Previous Tests: UPSC CAPF questions

36 New Public Management philosophy stands forComplete privatization of commercial government undertakings1.Decentralizing working units2.Establishing short-term labour contracts instead of long-term agreements3.Select the correct answer using the codes below.

1 and 2 onlyA.2 and 3 onlyB.1 and 3 onlyC.1, 2 and 3D.

User Answer :Correct Answer : BAnswer Justification :

Justification: Statement 1: NPM emphasizes the concept that ideas used in theprivate sector must be successful in the public sector. But, it does not advocatecomplete privatization of governmental functions.

Statement 2: This point suggest that it is more appropriate to shift from a unifiedmanagement system to a decentralized system in which managers gain flexibilityand are not limited to agency restrictions.

INSIG

HTS IAS

Page 25: Test 15 Solutions

TEST - 15

(C) Insights Active Learning. | All rights reserved. www.insightsias.com 25

Statement 3: It focuses on the necessity to establish short-term labor contracts,develop corporate plans, performance agreements and mission statements.

Q Source: Revision Previous Tests: UPSC CAPF questions

37 Consider the following statements.State Legislatures can make laws on matters enumerated in the Residuary list in1.special circumstances.State Legislatures cannot make laws on those matters enumerated in the Concurrent2.List on which a Central legislation already exists.Which of the above is/are correct?

1 onlyA.2 onlyB.Both 1 and 2C.NoneD.

User Answer :Correct Answer : DAnswer Justification :

Justification: Statement 1: The Parliament has exclusive powers to make laws withrespect to any of the matters enumerated in the Union List and Residuary list.

Statement 2: Both, the Parliament and state legislature can make laws with respectto any of the matters enumerated in the Concurrent List even if a Central law exists.But, laws made by state should not contravene Central laws.

Q Source: Chapter 14: Indian Polity: M Laxmikanth

38 Ricardian equivalence in economics refers to the notion thatTaxation and borrowing are equivalent means of financing expenditureA.Fiscal deficit is an unsustainable exercise.B.Investments and savings must be equal in a financial year in a closedC.economy.A country cannot maintain balance of payments if it wishes to lead inD.international trade

User Answer :Correct Answer : AAnswer Justification :

Learning: As per Ricardo, when the government increases spending by borrowing

INSIG

HTS IAS

Page 26: Test 15 Solutions

TEST - 15

(C) Insights Active Learning. | All rights reserved. www.insightsias.com 26

today, which will be repaid by taxes in the future, it will have the same impact onthe economy as an increase in government expenditure that is financed by a taxincrease today.

It has often been argued that 'debt does not matter because we owe it to ourselves'.This is because although there is a transfer of resources between generations,purchasing power remains within the nation.

However, any debt that is owed to foreigners involves a burden since we have tosend goods abroad corresponding to the interest payments.

Q Source: Chapter on Public Finance: 12th NCERT Macroeconomics:

39 According to Buddhist philosophyThere is nothing permanent in the world.1.Sorrow is intrinsic to human existence.2.Either extreme penance or extreme self-indulgence will open the doors to salvation.3.Human beings are unique in having a soul unlike other living beings.4.Select the correct answer using the codes below.

1 and 3 onlyA.2, 3 and 4 onlyB.1 and 2 onlyC.1, 3 and 4 onlyD.

User Answer :Correct Answer : CAnswer Justification :

Justification: Statement 1 and 4: According to Buddhist philosophy, the world istransient (anicca) and constantly changing; it is also soulless (anatta) as there isnothing permanent or eternal in it.

Statement 2 and 3: Within this transient world, sorrow is intrinsic to humanexistence. It is by following the path of moderation between severe penance andself-indulgence that human beings can rise above these worldly troubles.

Q Source: Revision of previous tests: Section on Buddhism: TamilNadu 11thStandard History Textbook

40 Which of the following is/are the federal features of the Constitution?The Constitution is written and not easily amendable.1.

INSIG

HTS IAS

Page 27: Test 15 Solutions

TEST - 15

(C) Insights Active Learning. | All rights reserved. www.insightsias.com 27

State governments derive authority from the Centre.2.Equal representation of all states in Rajya Sabha3.Select the correct answer using the codes below.

1 onlyA.2 and 3 onlyB.1 and 3 onlyC.1 and 2 onlyD.

User Answer :Correct Answer : AAnswer Justification :

Learning: The division of powers established by the Constitution as well as thesupremacy of the Constitution can be maintained only if the method of itsamendment is rigid. Hence, the Constitution is rigid to the extent that thoseprovisions which are concerned with the federal structure. So, 1 is correct.

State governments derive their authority from the constitution, not the Centre. So, 2is wrong.

The states are given representation in the Rajya Sabha on the basis of population.Hence, the membership varies from 1 to 31. So, 3 is wrong.

Q Source: Chapter 13: Indian Polity: M Laxmikanth

41 Which of the following is/are NOT mature Harappan sites?Balakot1.Rangpur2.Ganweriwala3.Digona4.Select the correct answer using the codes below.

1 and 2 onlyA.4 onlyB.3 and 4 onlyC.1, 2 and 3 onlyD.

User Answer :Correct Answer : BAnswer Justification :

Learning:The image has been lifted from the Q source.

If you are unable to see the image, refer to the Q source.

INSIG

HTS IAS

Page 28: Test 15 Solutions

TEST - 15

(C) Insights Active Learning. | All rights reserved. www.insightsias.com 28

Q Source:Revision Previous Tests Syllabus: Chapter 1: 12th NCERT: Part-I

42 Very small animals are rarely found in the Polar Regions. Which of the following canbe the most appropriate reason for this?

Small animals have a larger surface area relative to their volume and loseA.body heat very fast.Very Small animals do not perform any metabolic activity which is necessaryB.to survive in the Polar Regions.A large number of predators in Polar Regions prey on small animals.C.All of (a), (b) and (c)D.

User Answer :Correct Answer : AAnswer Justification :

Justification: Thermoregulation is energetically expensive for many organisms.This is particularly true for small animals like shrews and humming birds.

Heat loss or heat gain is a function of surface area. Since small animals have a largersurface area relative to their volume, they tend to lose body heat very fast when it iscold outside; then they have to expend much energy to generate body heat throughmetabolism.

INSIG

HTS IAS

Page 29: Test 15 Solutions

TEST - 15

(C) Insights Active Learning. | All rights reserved. www.insightsias.com 29

This is the main reason why very small animals are rarely found in Polar Regions.

Q Source: Revision Previous Tests Syllabus: 12th NCERT: Biology

43 Which of the following show the implementation of Directive Principles of StatePolicy?

Establishment of Central Administrative Tribunal1.Enactment of Legal Services Authorities Act2.73rd Amendment Act to the Constitution3.Implementation of land reforms4.Select the correct answer using the codes below.

1, 3 and 4 onlyA.2, 3 and 4 onlyB.1 and 4 onlyC.1, 2, 3 and 4D.

User Answer :Correct Answer : BAnswer Justification :

Learning: Statement 1: It addresses the service related grievances of centralgovernment employees. It does not implement any DPSP.

Statement 2: The Legal Services Authorities Act (1987) has established a nation-wide network to provide free and competent legal aid to the poor and to organise lokadalats for promoting equal justice.

Statement 3: Three-tier panchayati raj system (at village, taluka and zila levels) hasbeen introduced to translate into reality Gandhiji's dream of every village being arepublic.

Statement 4: It reduces inequality and leads to welfare of weaker sections of thepopulation.

Q Source: Chapter 8: Indian Polity: M Laxmikanth

44 Which of the following features of human body prevent entry or growth of harmfulmicrobes in the body?

Skin1.Acid in the stomach2.Saliva3.

INSIG

HTS IAS

Page 30: Test 15 Solutions

TEST - 15

(C) Insights Active Learning. | All rights reserved. www.insightsias.com 30

Eye tears4.Mucus coating lining the respiratory tract5.Select the correct answer using the codes below.

1, 2 and 5 onlyA.4 and 5 onlyB.1, 3 and 4 onlyC.1, 2, 3, 4 and 5D.

User Answer :Correct Answer : DAnswer Justification :

Learning: Innate immunity is non-specific type of defence, which is present at thetime of birth. This is accomplished by providing different types of barriers to theentry of the foreign agents into our body.

Innate immunity consists of four types of barriers.

These are -

Physical barriers: Skin on our body is the main barrier which prevents entryof the micro-organisms. Mucus coating of the epithelium lining therespiratory, gastrointestinal and urogenital tracts also help in trappingmicrobes entering our body.Physiological barriers: Acid in the stomach, saliva in the mouth, tears fromeyes-all prevent microbial growth.Cellular barriers: Certain types of leukocytes (WBC) of our body preventmicrobial growth.

Q Source: Revision Previous Tests Syllabus: 12th NCERT Biology

45 The Parliament cannot amend those provisions which form the 'basic structure' of theConstitution. This was ruled by the Supreme Court in the

Olga Tellis Case Vs. State of MaharashtraA.Bhikaji Vs. Union of India caseB.Kesavananda Bharati vs. State of KeralaC.Shankari Prasad CaseD.

User Answer :Correct Answer : CAnswer Justification :

Learning: Article 368 of the Constitution deals with the powers of Parliament to

INSIG

HTS IAS

Page 31: Test 15 Solutions

TEST - 15

(C) Insights Active Learning. | All rights reserved. www.insightsias.com 31

amend the Constitution and its procedure.

It states that the Parliament may, in exercise of its constituent power, amend by wayof addition, variation or repeal any provision of the Constitution in accordance withthe procedure laid down for the purpose.

But, if the amendment alters the basic structure, for e.g. Judicial review,Parliamentary democracy, Rule of Law, Secularism etc. then the amendment can bedeclared ultra vires by the SC.

Q Source: Chapter 10: Indian Polity: M Laxmikanth

46 The Tamil society witnessed a great change during the Pallava period. Which of thefollowing about the Pallava period is correct?

The caste system became rigid.1.The period witnessed the rise of Saivism and Vaishnavism.2.Buddhism and Jainism declined.3.Select the correct answer using the codes below.

1 and 2 onlyA.2 and 3 onlyB.1 and 3 onlyC.1, 2 and 3D.

User Answer :Correct Answer : DAnswer Justification :

Learning: The Brahmins occupied a high place in the society. They were givenland-grants by the kings and nobles. They were also given the responsibility oflooking after the temples.

The Saiva Nayanmars and the Vaishnava Alwars contributed to the growth ofSaivism and Vaishnavism. This is known as the Bakthi Movement.

They composed their hymns in the Tamil language. These hymns revealed theimportance of devotion or Bakthi. The construction of temples by the Pallava kingspaved the way for the spread of these two religions.

Q Source:Revision Previous Tests Syllabus: 11th TamilNadu History Textbook

47 Consider the following statements. Assertion (A): Marine sector contributes more to

INSIG

HTS IAS

Page 32: Test 15 Solutions

TEST - 15

(C) Insights Active Learning. | All rights reserved. www.insightsias.com 32

fishery production than inland sources. Reason (R): India has a long coastline with a largecoastal community involved in fishing. In the context of the above, which of these iscorrect?

A is correct, and R is an appropriate explanation of A.A.A is correct, but R is not an appropriate explanation of A.B.A is incorrect, but R is correct.C.Both A and R are correct independently.D.

User Answer :Correct Answer : CAnswer Justification :

Justification: The total fish production during 2013-14(Provisional) is registered9.58 million metric tonnes, with a contribution of 6.14 million metric tonnes frominland sector and 3.44 million metric tonnes from marine sector. So, A is wrong.

R is correct as the fisheries sector is a source of livelihood for over 14.49 millionpeople engaged fully, partially or in subsidiary activities related to the sector.

Q Source: Revision Previous Tests Syllabus: 11th NCERT: Indian EconomicDevelopment

48 Which of the following is/are the implications of promoting organic farming?International demand for organic food can be fulfilled.1.Organic farming is more labour intensive and suits a labour surplus economy India.2.It will give a fillip to the Genetic Engineering sector.3.It will make farming expensive as it requires costlier inputs.4.Select the correct answer using the codes below.

1 and 2 onlyA.1 and 4 onlyB.2 and 3 onlyC.1, 2 and 3D.

User Answer :Correct Answer : AAnswer Justification :

Justification:Statement 1: Organic agriculture also generates incomes throughinternational exports as the demand for organically grown crops is on a rise. Studiesacross countries have shown that organically grown food has more nutritional valuethan chemical farming thus providing us with healthy foods

Statement 2: Since organic farming requires more labour input than conventional

INSIG

HTS IAS

Page 33: Test 15 Solutions

TEST - 15

(C) Insights Active Learning. | All rights reserved. www.insightsias.com 33

farming, India will find organic farming an attractive proposition being a laboursurplus economy.

Statement 3: It does not use GM crops.

Statement 4: Organic agriculture offers a means to substitute costlier agriculturalinputs (such as HYV seeds, chemical fertilisers, pesticides etc.) with locallyproduced organic inputs that are cheaper and thereby generate good returns oninvestment.

Q Source: Revision Previous Tests Syllabus: 11th NCERT: Indian EconomicDevelopment

49 To get just and humane conditions for work and maternity relief is aFundamental RightA.Legal RightB.Directive Principle of State PolicyC.Executive DecreeD.

User Answer :Correct Answer : CAnswer Justification :

Learning: Some of the labour related DPSPs are:

To make provision for just and humane conditions for work and maternityrelief (Article 42).To secure a living wage, a decent standard of life and social and culturalopportunities for all workers (Article 43).To take steps to secure the participation of workers in the management ofindustries (Article 43 A).

Q Source:Chapter 8: Indian Polity: M Laxmikanth

50 Which of the following Directive Principles did NOT exist in the unamended originalconstitution?

To protect and improve the environment and to safeguard forests and wild life1.To secure for all citizens a uniform civil code throughout the country2.To secure the right to adequate means of livelihood for all citizens3.Select the correct answer using the codes below.

1 and 2 onlyA.

INSIG

HTS IAS

Page 34: Test 15 Solutions

TEST - 15

(C) Insights Active Learning. | All rights reserved. www.insightsias.com 34

2 and 3 onlyB.1 onlyC.1, 2 and 3D.

User Answer :Correct Answer : CAnswer Justification :

Learning: The 42nd Amendment Act of 1976 added four new Directive Principlesto the original list. They require the State:

To secure opportunities for healthy development of children (Article 39).To promote equal justice and to provide free legal aid to the poor (Article 39A).To take steps to secure the participation of workers in the management ofindustries (Article43 A).To protect and improve the environment and to safeguard forests and wildlife (Article 48 A).The 97th Amendment Act of 2011 added a new Directive Principle relating toco-operative societies. It requires the state to promote voluntary formation,autonomous functioning, democratic control and professional management ofco-operative societies (Article 43B).

Q Source: Chapter 8: Indian Polity: M Laxmikanth

51 The Wholesale Price Index (WPI) is released byDepartment of Finance, Ministry of FinanceA.Central Statistical Organization, Ministry of Statistics and ProgrammeB.ImplementationOffice of the Economic Adviser, Department of Industrial Policy &C.PromotionOffice of the Governor, Reserve Bank of IndiaD.

User Answer :Correct Answer : CAnswer Justification :

Learning:WPI is an important statistical indicator, as various policy decisions ofthe

Government, like inflation management, monitoring of prices of essentialcommodities etc., are based on it.

INSIG

HTS IAS

Page 35: Test 15 Solutions

TEST - 15

(C) Insights Active Learning. | All rights reserved. www.insightsias.com 35

It is one of the key variables for monetary policy changes by the Reserve Bank ofIndia. In addition to its role as a policy variable, WPI is also used by variousdepartments for arriving at the escalation costs of various contracts.

Headline inflation in India is measured in terms of Wholesale Price Index (WPI).

Q Source: Chapter 7: Indian Economy - Ramesh Singh

52 The Vesara style reached its culmination underRashtrakutas and HoysalasA.ChalukyasB.SatvahanasC.Imperial CholasD.

User Answer :Correct Answer : AAnswer Justification :

Learning: The Chalukyas were great patrons of art. They developed the vesara stylein the building of structural temples.

The structural temples of the Chalukyas exist at Aihole, Badami and Pattadakal.

Q Source: Revision Previous Tests Syllabus: 11th TamilNadu History Textbook

53 The Parliament can enact laws to implement which of the following fundamental dutiesunder Part IVA of the Constitution?

Penalty for not filing tax returns1.Penalty for not voting in elections2.Punishment for spreading communal hatred3.Select the correct answer using the codes below.

1 and 2 onlyA.3 onlyB.1 and 3 onlyC.1, 2 and 3D.

User Answer :Correct Answer : BAnswer Justification :

Justification: Filing tax returns and voting in elections are not fundamental duties.So, 1 and 2 will be wrong.

INSIG

HTS IAS

Page 36: Test 15 Solutions

TEST - 15

(C) Insights Active Learning. | All rights reserved. www.insightsias.com 36

It is a fundamental duty to promote harmony and the spirit of common brotherhoodamongst all the people of India transcending religious boundaries. Law can beenacted to make the provision legally effective.

Q Source:Chapter 9: Indian Polity: M Laxmikanth

54 Which of these is/are NOT Fundamental Duties of an Indian citizen?To properly nourish his/her children.1.To safeguard national property.2.To defend the country and render national service when called upon.3.Select the correct answer using the codes below.

1 and 2 onlyA.3 onlyB.1 onlyC.2 and 3 onlyD.

User Answer :Correct Answer : CAnswer Justification :

Justification:To provide opportunities for education to his child or ward betweenthe age of six and fourteen years is a fundamental duty. Nourishment, health,happiness etc don't come in it. So, 1 is wrong.

If there was an option like respect the elderly, you can tickmark it right, sincevaluing and preserving the rich heritage of the country's composite culture is afundamental duty.

Q Source: Chapter 9: Indian Polity: M Laxmikanth

55 The Union Cabinet Committee on Economic Affairs (CCEA) has approved PradhanMantri Ujjwala Yojana (PMUY) for

Providing free of cost LPG connections to women from BPL Households.A.Giving subsidized LED lights to power consumersB.Providing scholarships to BPL students for higher educationC.Giving free electricity connections in Naxal affected blocksD.

User Answer :Correct Answer : AAnswer Justification :

Learning: The PMUY seeks to empowering women and protecting their health by

INSIG

HTS IAS

Page 37: Test 15 Solutions

TEST - 15

(C) Insights Active Learning. | All rights reserved. www.insightsias.com 37

shifting them from traditional cooking based on unclean cooking fuels or on fossilfuels that have serious health hazards to clean cooking gas.

Financial support of 1600 rupees for each LPG connection will be provided to theBPL households. The identification of eligible BPL families will be made inconsultation with the State Governments and the Union Territories. It will beimplemented over three years' time frame namely in the FY 2016-17, 2017-18 and2018-19.

Q Source:http://www.newindianexpress.com/states/odisha/Ujjwala-Yojana-to-Put-Odisha-on-Sticky-Wicket/2016/03/12/article3322905.ece

56 Consider the following statements.Assertion (A): The president must give her assent toa Constitutional amendment bill passed by due process.Reason (R): The Constitutionmakers did not give the office of the President any responsibility with regard to protectingthe constitution.In the context of the above, which of these is correct?

A is correct, and R is an appropriate explanation of A.A.A is correct, but R is not an appropriate explanation of A.B.A is incorrect, but R is correct.C.A is correct, but R is incorrect.D.

User Answer :Correct Answer : DAnswer Justification :

Justification:The president must give his assent to the bill. He can neither withholdhis assent to the bill nor return the bill for reconsideration of the Parliament.

The 24th Constitutional Amendment Act of 1971 made it obligatory for thePresident to give his assent to a constitutional Amendment Bill.

Before this he could treat a constitutional amendment bill just as an ordinary bill andcould use his veto powers. So, R is wrong.

Q Source: Chapter 10: Indian Polity: M Laxmikanth

57 Which of the following provisions of the Constitution can be amended by a simplemajority in the Parliament?

Conferment of more jurisdiction on the Supreme Court1.Delimitation of constituencies2.

INSIG

HTS IAS

Page 38: Test 15 Solutions

TEST - 15

(C) Insights Active Learning. | All rights reserved. www.insightsias.com 38

Directive Principles of State Policy3.Election of the President and its manner4.Select the correct answer using the codes below.

1 and 2 onlyA.3 and 4 onlyB.1 onlyC.1, 2, 3 and 4D.

User Answer :Correct Answer : AAnswer Justification :

Justification: Statement 3 requires special majority.

Statement 4 requires Special Majority of Parliament and Consent of States.

Other important provisions that can be changed by simple majority of Parliamentare:

Use of official language.Citizenship-acquisition and termination.Elections to Parliament and state legislatures.Delimitation of constituencies.Union territories.Fifth Schedule-administration of scheduled areas and scheduled tribes.Sixth Schedule-administration of tribal areas.

Q Source:Chapter 10: Indian Polity: M Laxmikanth

58 The lower house of the Parliament, Lok Sabha, can be dissolved byThe President on recommendation of the Prime MinisterA.The Prime Minister on the advice of the Council of MinistersB.The Supreme Court on an inquiry conducted by a Parliamentary CommitteeC.The Speaker on the recommendations of a Joint Committee of both housesD.

User Answer :Correct Answer : AAnswer Justification :

Learning:The prime minister can advise the President to dissolve the Lok Sabhabefore the expiry of its term and hold fresh elections.

This means that the executive enjoys the right to get the legislature dissolved in a

INSIG

HTS IAS

Page 39: Test 15 Solutions

TEST - 15

(C) Insights Active Learning. | All rights reserved. www.insightsias.com 39

parliamentary system.

Q Source:Chapter 12: Indian Polity: M Laxmikanth

59 Sri Lanka is separated from India by a narrow channel of sea formed by theGulf of MannarA.Palk StraitB.Kandy IsthmusC.Hambantota BridgeD.

User Answer :Correct Answer : BAnswer Justification :

Learning: It is a strait between Tamil Nadu and Mannar district of the NorthernProvince of Sri Lanka.

It connects the Bay of Bengal in the northeast with the Palk Bay and thence with theGulf of Mannar in the southwest.

It is studded at its southern end with a chain of low islands and reef shoals that arecollectively called Adam's Bridge, since ages it is popularly known in HinduMythology as "Ram Setu".

Q Source: Chapter 1: India Yearbook 2016

60 'Inflationary Gap' refers to theExcess demand of goods over production in the economyA.Large difference in inflation in the preceding and succeeding yearB.Inflation that does not trigger an increase in GDPC.Distance between the current level of real GDP and full employment realD.GDP

User Answer :Correct Answer : DAnswer Justification :

Learning: Real GDP is nominal GDP adjusted for inflation.

The inflationary gap is so named because the relative increase in real GDP causes aneconomy to increase its consumption, which causes prices to rise in the long run.

INSIG

HTS IAS

Page 40: Test 15 Solutions

TEST - 15

(C) Insights Active Learning. | All rights reserved. www.insightsias.com 40

The main cause of the gap is considered to be expansionary monetary policiescarried out by the government.

An inflationary gap is a signal that the economy is in the boom part of the tradecycle, resources are being used over their capacity, factories are operating withincreasing average costs; wage rates increase because labour is used beyond normalhours at overtime pay rates.

Q Source: Chapter 7: Indian Economy - Ramesh Singh

61 The principle of collective responsibility is the bedrock principle of parliamentarygovernment. The principle implies that the

Lok Sabha can remove the council of ministers from office by passing a vote of no1.confidence.The Council of Ministers as a whole is bound by consensus.2.Which of the above is/are correct?

1 onlyA.2 onlyB.Both 1 and 2C.NoneD.

User Answer :Correct Answer : CAnswer Justification :

Learning: The ministers are collectively responsible to the Parliament in generaland to the Lok Sabha in particular (Article 75).

They act as a team, and swim and sink together. The principle of collectiveresponsibility implies that the Lok Sabha can remove the ministry (i.e., council ofministers headed by the prime minister) from office by passing a vote of noconfidence.

Members of the council of ministers are bound by consensus. The governmentcannot have two opinions on the same issue.

Q Source: Chapter 12: Indian Polity: M Laxmikanth

62 Consider the following with reference to the Gender related data as recorded by theCensus 2011.

It has recorded the highest ever sex ratio in India since independence.1.

INSIG

HTS IAS

Page 41: Test 15 Solutions

TEST - 15

(C) Insights Active Learning. | All rights reserved. www.insightsias.com 41

More number of females have become literate than males since Census 2001.2.Which of the above is/are correct?

1 onlyA.2 onlyB.Both 1 and 2C.NoneD.

User Answer :Correct Answer : BAnswer Justification :

Justification: Statement 1: Overall sex ratio at the national level increased by 7points since census 2001 to reach 943 at census 2011. This is the highest sex ratiorecorded since census 1961, not since independence. It is lower than the sex ratio in1951 (it was 946).

Statement 2: Literacy rate has gone up from 64.8 per cent in 2001 to 73.0 per centshowing an increase of 8.2 percentage points. It is encouraging to note that out of atotal of 202,810,720 literates added during the decade, females 104,660,657outnumber males 98,150,063.

Q Source: Chapter 1: India Yearbook 2016

63 The parliamentary system of government in India is largely based on the Britishparliamentary system. However the Indian system differs from the British in which of thefollowing?

India is a Republic whereas Britain is a constitutional Monarchy.1.Citizens other than MPs cannot be appointed as Ministers in India unlike in Britain.2.In Britain the prime minister should be a member of the Lower House unlike in3.India where he can be a member of any house.Select the correct answer using the codes below.

1 and 2 onlyA.2 and 3 onlyB.1 and 3 onlyC.1, 2 and 3 onlyD.

User Answer :Correct Answer : CAnswer Justification :

Justification: In Britain, the prime minister should be a member of the LowerHouse (House of Commons) of the Parliament. In India, the prime minister may be a

INSIG

HTS IAS

Page 42: Test 15 Solutions

TEST - 15

(C) Insights Active Learning. | All rights reserved. www.insightsias.com 42

member of any of the two Houses of Parliament

Usually, the members of Parliament alone are appointed as ministers in Britain. InIndia, a person who is not a member of Parliament can also be appointed as minister,but for a maximum period of six months.

Q Source: Chapter 12: Indian Polity: M Laxmikanth

64 Many early sculptors did not show the Buddha in human form - instead, they showedhis presence through symbols. Which of the following symbols have been used to representthe Buddha and his teachings?

Empty Seat1.Divine Light2.Wheel3.The Bodhi Tree4.Endless Knots5.Plain white cloth6.Select the correct answer using the codes below.

1, 2, 4 and 6 onlyA.2, 3 and 6 onlyB.1, 3, 4 and 5 onlyC.1, 2, 3, 4, 5 and 6D.

User Answer :Correct Answer : CAnswer Justification :

Learning: Mahayana and Vajrayana Buddhist art frequently makes use of aparticular set of eight auspicious symbols, ashtamangala, in domestic and public art.These symbols have spread with Buddhism to the art of many cultures, includingIndian, Tibetan, Nepalese, and Chinese art.

These symbols are:

Lotus flower. Representing purity and enlightenment.Endless knot, or, the Mandala. Representing eternal harmony.Golden Fish pair. Representing conjugal happiness and freedom.Victory Banner. Representing a victorious battle.Wheel of Dharma or Chamaru in Nepali Buddhism. Representing knowledge.Treasure Vase. Representing inexhaustible treasure and wealth.Parasol. Representing the crown, and protection from the elements.Conch shell. Representing the thoughts of the Buddha.

INSIG

HTS IAS

Page 43: Test 15 Solutions

TEST - 15

(C) Insights Active Learning. | All rights reserved. www.insightsias.com 43

Q Source: Revision of previous tests: Section on Buddhism: TamilNadu 11thStandard History Textbook

65 Which of the following trends in the working of Indian political system reflects itsfederal spirit?

Territorial disputes between states1.Demand for creation of new states2.Demand of the states for more financial grants from the Centre to meet their3.developmental needsEmergence of Regional parties and their contribution in national politics4.Opposition of the State to the Goods and Services Tax (GST)5.Select the correct answer using the codes below.

1, 2 and 3 onlyA.2, 4 and 5 onlyB.3, 4 and 5 onlyC.1, 2, 3, 4 and 5D.

User Answer :Correct Answer : DAnswer Justification :

Learning: Although the Constitution of India has created a strong Centralgovernment, it has not made the state governments weak and has not reduced themto the level of administrative agencies for the execution of policies of the Centralgovernment.

Indian federation has been described as as "a new kind of federation to meet India'speculiar needs".

It is clearly seen from the above examples. This implies that the state governmentsare autonomous units working under Constitutional framework in tandem with theCentre to achieve common goals.

Q Source: Chapter 13: Indian Polity: M Laxmikanth

66 The National Air Quality Index takes into account which of the following air pollutants?Particulate Matter1.Asbestos2.Ozone3.Lead4.Carbon dioxide5.

INSIG

HTS IAS

Page 44: Test 15 Solutions

TEST - 15

(C) Insights Active Learning. | All rights reserved. www.insightsias.com 44

Select the correct answer using the codes below.1, 3 and 4 onlyA.2, 3 and 4 onlyB.1 and 5 onlyC.1, 2, 3, 4 and 5D.

User Answer :Correct Answer : AAnswer Justification :

Learning: The National AQI is published for every month by CPCB along with anumerical value and a colour code which helps in comparing air pollution levels incities.

It is determined on the basis of concentration of 8 pollutants, including ParticulateMatter (PM 2.5, PM 10), sulphur dioxide (SO2), nitrogen dioxide (NO2), carbonmonoxide (CO), ozone (O3), ammonia (NH3) and lead (Pb).

The colour categories are classified into 6 categories depending upon numericalvalue as Good (0-50), Satisfactory (51-100), moderately polluted (101-200), Poor(201-300), Very poor (301-400) and Severe (401-500).

Q Source: Previous year Current affairs

http://pib.nic.in/newsite/PrintRelease.aspx?relid=110654

67 The Constitution empowers the Parliament to make laws on any matter enumerated inthe State List under which of the following extraordinary circumstances?

When the President exhorts the Council of Ministers to do so in national interest1.When the lower house passes a resolution to this effect by special majority2.When a National Emergency is in operation3.When the National Security Council recommends so in the interest of national4.securitySelect the correct answer using the codes below.

1 and 2 onlyA.3 onlyB.3 and 4 onlyC.1, 2, 3 and 4D.

User Answer :Correct Answer : BAnswer Justification :

INSIG

HTS IAS

Page 45: Test 15 Solutions

TEST - 15

(C) Insights Active Learning. | All rights reserved. www.insightsias.com 45

Learning: If the Rajya Sabha declares that it is necessary in the national interestthat Parliament should make laws on a matter in the State List, then the Parliamentbecomes competent to make laws on that matter. This cannot be declared by thePresident, Lok Sabha or the National Security Council. So, all statements except 3are wrong.

The Parliament can make laws on any matter in the State List for implementing theinternational treaties, agreements or conventions.

Q Source:Chapter 14: Indian Polity: M Laxmikanth

68 Consider the following statements about United Nations (UN) peacekeeping operations.UN maintains an independent peacekeeping army funded by contributions from1.member states.UN Security Council authorizes peacekeeping operations.2.Regional organizations can be authorized to undertake peacekeeping operations.3.Select the correct answer using the codes below.

1 and 2 onlyA.2 and 3 onlyB.2 onlyC.1 and 3 onlyD.

User Answer :Correct Answer : BAnswer Justification :

Justification:Statement 1 and 3 : Most of these operations are established andimplemented by the United Nations itself, with troops serving under UN operationalcontrol.

In these cases, peacekeepers remain members of their respective armedforces, and do not constitute an independent "UN army," as the UN does nothave such a force.In cases where direct UN involvement is not considered appropriate orfeasible, the Council authorizes regional organizations such as the NATO orcoalitions of willing countries to undertake peacekeeping or peace-enforcement tasks.

Statement 2: The UN Charter gives the UNSC the power and responsibility to takecollective action to maintain international peace and security. For this reason, theinternational community usually looks to the UNSC to authorize peacekeepingoperations through Chapter VI authorizations.

INSIG

HTS IAS

Page 46: Test 15 Solutions

TEST - 15

(C) Insights Active Learning. | All rights reserved. www.insightsias.com 46

Q Source: Current affairs + Revision Previous Tests: 12th NCERT: ContemporaryWorld Politics

69 Inflation in India is measured 'point-to-point'. It can mean thatReference dates for calculating inflation are from the preceding andA.succeeding month of the same yearReference dates for calculating inflation are from the preceding andB.succeeding quarter of the same yearThe same date is taken as reference for calculating inflation from theC.preceding and succeeding yearsAll of (a), (b), (c)D.

User Answer :Correct Answer : CAnswer Justification :

Justification: It means that the reference dates for the annual inflation is January 1to January 1 of two consecutive years (not for January 1 to December 31 of theconcerned year).

Similarly, the weekly rate of inflation is the change in one week reference being thetwo consecutive last days of the week (i.e., 5 p.m. of two Fridays in India).

Q Source:Chapter 7: Indian Economy - Ramesh Singh

70 Which of the following are extra-constitutional devices to promote cooperation andcoordination between the Centre and the states?

Zonal Councils1.Inter-State Councils2.North-Eastern Council3.Central Council of Indian Medicine4.Select the correct answer using the codes below.

1 and 3 onlyA.2 and 4 onlyB.1, 3 and 4 onlyC.1, 2, 3 and 4 onlyD.

User Answer :Correct Answer : CAnswer Justification :

Learning: Subject specific Inter-State Councils set up under Article 263 of the

INSIG

HTS IAS

Page 47: Test 15 Solutions

TEST - 15

(C) Insights Active Learning. | All rights reserved. www.insightsias.com 47

Constitution of India

Central Council of HealthCentral Council for Local Government and Urban DevelopmentRegional Councils for Sales Tax and State Excise Duties

Inter-State Coordination Mechanism set up outside the framework of Article 263 ofthe Constitution of India

Planning Commission of India (Now abolished)National Development CouncilNational Integration CouncilCentral Advisory Board of EducationCentral Council for Research in Ayurveda & SiddhaCentral Council for Research in HomoeopathyCentral Council for Research in Yoga & NaturopathyLabour Conference

Q Source: Chapter 14: Indian Polity: M Laxmikanth

71 From the various judgements of the Supreme Court, which of the following haveemerged as elements of 'basic structure' of the Constitution?

Separation of powers between the legislature, the executive and the judiciary1.Harmony and balance between Fundamental Rights and Directive Principles2.Supremacy of the Constitution3.Principle of reasonableness4.Welfare state and ideals of Socio-economic Justice5.Select the correct answer using the codes below.

1, 2 and 3 onlyA.1, 3 and 5 onlyB.2, 4 and 5 onlyC.1, 2, 3, 4 and 5D.

User Answer :Correct Answer : DAnswer Justification :

Learning: Apart the above, these are the elements:

Sovereign, democratic and republican nature of the Indian politySecular character of the ConstitutionFederal character of the Constitution

INSIG

HTS IAS

Page 48: Test 15 Solutions

TEST - 15

(C) Insights Active Learning. | All rights reserved. www.insightsias.com 48

Unity and integrity of the nationJudicial reviewFreedom and dignity of the individualParliamentary systemRule of lawPrinciple of equalityFree and fair electionsIndependence of JudiciaryLimited power of Parliament to amend the ConstitutionEffective access to justicePowers of the Supreme Court under Articles 32, 136, 141 and 142

Q Source:Chapter 11: Indian Polity: M Laxmikanth

72 Statutory grants given under Article 275 areGrants given to states in need of financial assistance and not to every stateA.Grants made for any public purpose by the Centre to the StatesB.Temporary grants given by the Centre to the states which the states mustC.repay in due timeGrants recommended by erstwhile Planning CommissionD.

User Answer :Correct Answer : AAnswer Justification :

Learning: Article 275 empowers the Parliament to make grants to the states whichare in need of financial assistance and not to every state. Also, different sums maybe fixed for different states. These sums are charged on the Consolidated Fund ofIndia every year.

Apart from this general provision, the Constitution also provides for specific grantsfor promoting the welfare of the scheduled tribes in a state or for raising the level ofadministration of the scheduled areas in a state including the State of Assam.

The statutory grants under Article 275 (both general and specific) are given to thestates on the recommendation of the Finance Commission.

Q Source:Chapter 14: Indian Polity: M Laxmikanth

73 Every year International Women's Day is being observed on 8th March across the globeto highlight the achievements of women while calling for greater equality. What is the

INSIG

HTS IAS

Page 49: Test 15 Solutions

TEST - 15

(C) Insights Active Learning. | All rights reserved. www.insightsias.com 49

theme for 2016 as declared by the United Nations?Planet 50-50 by 2030: Step It Up for Gender EqualityA.World Free of Violence Against WomenB.Empower Rural Women, End Poverty and HungerC.Women in Decision-makingD.

User Answer :Correct Answer : AAnswer Justification :

Learning:Themes in options other than (a) are previous year themes.

The day is an official holiday in countries like Afghanistan, Angola, Armenia, Nepal(women only) etc.

The earliest celebration was held as a Socialist political event in 1909. It blended theculture of many countries primarily in Europe, especially those in the Soviet Bloc.

Declared a national holiday in the Soviet Union in 1917, it spread to other nearbycountries. It is now celebrated in many Eastern countries.

Q Source:International Women's Day

74 The President can proclaim a national emergency whenCabinet gives a written recommendation to the PresidentA.The Parliament passes a resolution by simple majority to this effectB.The President is individually satisfied that it is in the best national interestC.The National Security Council recommends it to the PresidentD.

User Answer :Correct Answer : AAnswer Justification :

Learning:The emergency can be declared only on the concurrence of the cabinetand not merely even on the advice of the prime minister or Home Minister.

The proclamation of Emergency should be approved by both the Houses ofParliament within one month from the date of its issue. So, (b) is wrong.

In 1975, the then Prime Minister, Indira Gandhi advised the president to proclaimemergency without consulting her cabinet.

The cabinet was informed of the proclamation after it was made, as a fait accompli.

INSIG

HTS IAS

Page 50: Test 15 Solutions

TEST - 15

(C) Insights Active Learning. | All rights reserved. www.insightsias.com 50

The 44th Amendment Act of 1978 introduced this safeguard to eliminate anypossibility of the prime minister alone taking a decision in this regard.

Q Source:Chapter 16: Indian Polity: M Laxmikanth

75 During a national emergencyState governments are suspended and Centre takes the control of the states.1.Parliament becomes empowered to make laws on any subject mentioned in the State2.List.Which of the above is/are correct?

1 onlyA.2 onlyB.Both 1 and 2C.NoneD.

User Answer :Correct Answer : BAnswer Justification :

Justification:Statement 1: The Centre becomes entitled to give executive directionsto a state on 'any' matter. Thus, the state governments are brought under thecomplete control of the Centre, though they are not suspended.

Statement 2: Although the legislative power of a state legislature is not suspended, itbecomes subject to the overriding power of the Parliament.

Thus, the normal distribution of the legislative powers between the Centre and statesis suspended, though the state legislatures are not suspended. In brief, theConstitution becomes unitary rather than federal.

Q Source:Chapter 16: Indian Polity: M Laxmikanth

76 Phillips Curve shows the trade-off betweenGDP growth and population growthA.Inflation and unemploymentB.Environmental degradation and economic inequalityC.Real GDP growth and inflationD.

User Answer :Correct Answer : BAnswer Justification :

INSIG

HTS IAS

Page 51: Test 15 Solutions

TEST - 15

(C) Insights Active Learning. | All rights reserved. www.insightsias.com 51

Learning:As per the curve there is a 'trade off' between inflation and unemploymenti.e. an inverse relationship between them. The curve suggests that lower theinflation, higher the unemployment and higher the inflation, lower theunemployment.

During 1960s, this idea was among the most important theories of the moderneconomists.

However, economists later contested this curve and suggested some modifications toit.

Q Source:Chapter 7: Indian Economy - Ramesh Singh

77 The presidential proclamation imposing President's Rule is subject to judicial reviewwas declared by the Supreme Court in

D.C. Wadhwa CaseA.S.R. Bommai CaseB.Rameshwar Prasad CaseC.M.C. Mehta CaseD.

User Answer :Correct Answer : BAnswer Justification :

Learning: The Supreme Court declared in D. C. Wadhwa v. State of Bihar (1986),that it is unconstitutional to repromulgate ordinances (in certain conditions).

The case of M.C. Mehta v. Union of India originated in the aftermath of oleum gasleak from Shriram Food and Fertilisers Ltd. complex at Delhi.

Rameshwar Prasad case was related to dissolution of Bihar Assembly.

Q Source:Chapter 16: Indian Polity: M Laxmikanth

78 The song Vande Mataram, composed in Sanskrit by Bankimchandra Chatterji, was asource of inspiration to the people in their struggle for freedom. It was first sung at

1929 Purna Swaraj declaration session at LahoreA.Calcutta during the mutiny of 1857B.1896 session of the Indian National CongressC.Swadeshi movement 1905 after partition of BengalD.

User Answer :

INSIG

HTS IAS

Page 52: Test 15 Solutions

TEST - 15

(C) Insights Active Learning. | All rights reserved. www.insightsias.com 52

Correct Answer : CAnswer Justification :

Learning:It has an equal status with Jana-gana-mana, our national anthem.

Jana-gana-mana was first sung in 1911 at the Kolkata Session of the Indian NationalCongress.

The English translation of Vande Matram stanzas was rendered by Sri Aurobindo.

Q Source:National Song: Chapter 2: India Yearbook 2016

79 An unmanned spacecraft named ExoMars 2016 was launched jointly Europe and Russiato

Search for biosignatures on Mars in past or present.1.Study the outer atmosphere of Mars and its impact on earth’s long-term climate.2.Which of the above is/are correct?

1 onlyA.2 onlyB.Both 1 and 2C.NoneD.

User Answer :Correct Answer : AAnswer Justification :

Learning:Its primary Goal is to address the question of whether life has ever existedon Mars.

Its sole purpose is to search (trace) evidence of methane (CH4) and otheratmospheric gases that could be signatures of active biological or geologicalprocesses on Mars.

For the first time it will provide new insights into the role of electric forces on dustlifting, the trigger for dust storms on the Martian surface.

QSource:http://www.thehindu.com/todays-paper/tp-national/exomars-giant-nose-to-sniff-out-life-on-mars/article8349939.ece

80 Consider the following about the missile Agni-I.

INSIG

HTS IAS

Page 53: Test 15 Solutions

TEST - 15

(C) Insights Active Learning. | All rights reserved. www.insightsias.com 53

It is a nuclear capable surface-to-surface ballistic missile.1.It can travel faster than the speed of sound.2.Its sophisticated navigation system can navigate the missile to hit target accurately.3.Select the correct answer using the codes below.

1 and 2 onlyA.1 onlyB.2 and 3 onlyC.1, 2 and 3D.

User Answer :Correct Answer : DAnswer Justification :

Learning:Agni-I is the first missile of the Agni series launched in 1983 by theDRDO as part of Integrated Guided Missile Development Programme (IGMDP) inIndia.

Its testing is conducted as part of training exercise by the Strategic Forces Command(SFC) of Indian Army.

It is powered by both solid and liquid propellants and can be fired from road and railmobile launchers.

The Agni I missile already has been inducted into armed forces and claimed to be apart of the India's minimum credible deterrence under No first to use policy.

QSource:http://www.thehindu.com/news/national/india-testfires-nuclearcapable-agnii/article8351534.ece

81 China's Supreme Court has decided to set up its own International Maritime JudicialCentre (IMJC) to handle territorial disputes and protect its sea rights. However,International maritime disputes between countries are usually brought before the

International Court of Justice (ICJ)A.International maritime organisation (IMO)B.United Nations Security Council (UNSC)C.International Chamber of Shipping (ICM)D.

User Answer :Correct Answer : AAnswer Justification :

Learning:Presently China is locked in disputes with its neighbours over claims in

INSIG

HTS IAS

Page 54: Test 15 Solutions

TEST - 15

(C) Insights Active Learning. | All rights reserved. www.insightsias.com 54

the resource-rich South China Sea.

The tensions have risen recently over China's aggressive land reclamation continuesto build artificial islands, airport runways and facilities on disputed reefs.

This move will help China to bolster its claims in the disputed South and East Chinaseas and also help it become a maritime power.

QSource:http://www.business-standard.com/article/news-ians/china-to-build-international-maritime-judicial-centre-116031300070_1.html

82 The Government of India is celebrating April 21 every year as 'Civil Services Day'. 21stApril has been chosen as date because

Sardar Vallabhbhai Patel addressed the first batch of Indian AdministrativeA.Services (IAS) officers on this date.Indian Covenanted Civil Services were created by the British on this dateB.First batch of Civil servants of Independent India graduated on this date.C.Pandit Nehru chose this date based on the first International meeting of CivilD.Servants.

User Answer :Correct Answer : AAnswer Justification :

Learning: On the occasion, civil servants are awarded by the Prime Minister fortheir excellent work done in the field of public administration.

It is celebrated as an occasion for the civil servants to rededicate themselves to thecause of citizens and renew their commitment to public service and excellence inwork.

First such function was held in 2006.

Q Source:Chapter 3: India Yearbook 2016

83 Wholesale Price Index (WPI) indicates price rise in which of the following categories?Fuel1.Power2.Gold and Silver3.

INSIG

HTS IAS

Page 55: Test 15 Solutions

TEST - 15

(C) Insights Active Learning. | All rights reserved. www.insightsias.com 55

Some consumer durables4.Select the correct answer using the codes below.

1 and 2 onlyA.4 onlyB.1, 2 and 4 onlyC.1, 2, 3 and 4D.

User Answer :Correct Answer : DAnswer Justification :

Justification:There are three major groups in WPI - Primary Articles, Fuel & Powerand Manufacture Products. Highest weightage has been accorded to themanufactured products.

Consumer durables are a part of the manufactured products category.

The WPI series base year, article composition and weightage has also been revisedfew times.

The Office of the Economic Adviser undertook the work relating to revision of theexisting series of WPI, which not only addressed the issue of change in base year,but also revised the entire commodity basket and the weighting diagram so as tobetter reflect the price trends in economy.

Q Source:Chapter 7: Indian Economy - Ramesh Singh

84 Consider the following about the Law Commission of India (LCI).It is a non-statutory and non-constitutional body.1.It is headed by the incumbent Union Law Secretary.2.It is constituted by the Parliament from time to time.3.Select the correct answer using the codes below.

1 onlyA.2 and 3 onlyB.1 and 3 onlyC.1, 2 and 3 onlyD.

User Answer :Correct Answer : AAnswer Justification :

Justification:Statement 2: It is usually headed by a retired Supreme Court judge orformer Chief Justice of a high court.

INSIG

HTS IAS

Page 56: Test 15 Solutions

TEST - 15

(C) Insights Active Learning. | All rights reserved. www.insightsias.com 56

Statement 3: It is constituted by the Union Government from time to time. The firstcommission was constituted in 1955 and since then various commissions were re-constituted every three years.

Q Source:Frequently in news

85 Consider the following statements.Assertion (A): In India, only the Central government can make developmental plans.1.Reason (R): 'Economic and social planning' falls under the Union List in the2.Seventh schedule of the Constitution.In the context of the above, which of these is correct?

A is correct, and R is an appropriate explanation of A.A.A is correct, but R is not an appropriate explanation of A.B.A is incorrect, but R is correct.C.Both A and R are incorrect.D.

User Answer :Correct Answer : DAnswer Justification :

Learning:'Economic and social planning' is a concurrent subject. Also, whileframing the 'Union', 'State' and 'Concurrent' list, allocating subjects and otherprovisions, the Constitution vests power in the Union to ensure co-ordinateddevelopment in essential fields of activity while preserving the initiative andauthority of the states in the spheres allotted to them.

Q Source:MAJOR OBJECTIVES OF PLANNING: Chapter 5: Indian Economy -Ramesh Singh

86 The applications of Indian Regional Navigational Satellite System (IRNSS) includeDisaster Management1.Vehicle tracking and fleet management2.Mapping3.Select the correct answer using the codes below.

1 and 2 onlyA.2 and 3 onlyB.2 onlyC.1, 2 and 3D.

User Answer :Correct Answer : DAnswer Justification :

INSIG

HTS IAS

Page 57: Test 15 Solutions

TEST - 15

(C) Insights Active Learning. | All rights reserved. www.insightsias.com 57

Learning:This satellite system aims to provide real-time data on the position ofobjects to aid road, air and maritime traffic. It will also provide mapping andtracking services.

IRNSS System consists of constellation of seven satellites of which three aregeostationary and four are non-geostationary.

It will also give facility for terrestrial navigation aid for hikers and travellers andvisual and voice navigation for drivers.

Q Source:Frequently in news

87 A constitutional amendment bill that amends major federal features can be passed bySpecial majority in each house of the ParliamentA.Absolute Majority in any one house of the ParliamentB.Simple majority in both house of Parliament as well as in majority of StateC.LegislaturesNone of the above is correct.D.

User Answer :Correct Answer : DAnswer Justification :

Learning:Ordinarily, the bill must be passed in each House by a special majority,that is, a majority (that is, more than 50 per cent) of the total membership of theHouse and a majority of two-thirds of the members of the House present and voting.

Each House must pass the bill separately. In case of a disagreement between the twoHouses, there is no provision for holding a joint sitting of the two Houses for thepurpose of deliberation and passage of the bill.

If the bill seeks to amend the federal provisions of the Constitution, it must also beratified by the legislatures of half of the states by a simple majority, that is, amajority of the members of the House present and voting. So, (d) is the only correctanswer.

Q Source:Chapter 10: Indian Polity: M Laxmikanth

88 The Parliament has passed National Waterways Bill, 2015 to clear way for theconversion of several water bodies across India into transport Inland waterways. Apartfrom rivers, which of the following can be said to comprise inland waterways?

INSIG

HTS IAS

Page 58: Test 15 Solutions

TEST - 15

(C) Insights Active Learning. | All rights reserved. www.insightsias.com 58

Lakes1.Canals2.Creeks3.Backwaters4.Select the correct answer using the codes below.

1, 2 and 4 onlyA.1 and 2 onlyB.3 and 4 onlyC.1, 2, 3 and 4D.

User Answer :Correct Answer : DAnswer Justification :

Learning:The National Waterways Bill, 2015 aims at declaring 106 additionalinland waterways as National Waterways in addition to the five existing NationalWaterways.

It repeals the existing five Acts that declare the existing 5 national waterways.These existing five national waterways are now covered under the new Bill.Thus the total number of national waterways number will go up to 111 fromexisting five national waterways.Inland waterways mainly comprise rivers, lakes, canals, creeks andbackwaters. It extends about 14,500 km across the country.However, potential of this mode of transport has not been fully exploited sofar.

Q Source:http://www.prsindia.org/billtrack/the-national-waterways-bill-2015-3776/

89 Consider the following about Convention on Conservation of Migratory Species (CMS).It is also known as Bonn Convention1.It was concluded under the aegis of the United Nations Environment Programme2.(UNEP).India is a party to the CMS.3.It covers only terrestrial and avian migratory species throughout their range.4.Select the correct answer using the codes below.

1 and 4 onlyA.2 and 3 onlyB.1, 2 and 3 onlyC.1, 2, 3 and 4D.

User Answer :

INSIG

HTS IAS

Page 59: Test 15 Solutions

TEST - 15

(C) Insights Active Learning. | All rights reserved. www.insightsias.com 59

Correct Answer : CAnswer Justification :

Learning:The CMS is the only global and UN-based intergovernmentalorganization established exclusively for the conservation and management ofterrestrial, aquatic and avian migratory species throughout their range.

It is an intergovernmental treaty, concluded under the aegis of the United NationsEnvironment Programme, concerned with the conservation of wildlife and habitatson a global scale.

Since the Convention's entry into force, its membership has grown steadily toinclude over 100 Parties from Africa, Central and South America, Asia, Europe andOceania.

You may check the species coverage here

https://en.wikipedia.org/wiki/Convention_on_the_Conservation_of_Migratory_Species_of_Wild_Animals#Species_Coverage

Q Source:India has recently signed ‘Raptor MoU’, a Memorandum ofUnderstanding (MoU) on conservation of birds of prey in Africa and Eurasia.

90 Consider the following about the inter-state council.Chief Ministers of all the states are its members.1.Five Ministers nominated by the Prime Minister are permanent invitees to the2.Council.The council is a recommendatory body3.The Council meets once biannually.4.Select the correct answer using the codes below.

1, 2 and 3 onlyA.2 and 4 onlyB.1, 3 and 4 onlyC.1, 2, 3 and 4 onlyD.

User Answer :Correct Answer : AAnswer Justification :

Learning:The council is a recommendatory body on issues relating to inter-state,Centre-state and Centre-union territories relations.

INSIG

HTS IAS

Page 60: Test 15 Solutions

TEST - 15

(C) Insights Active Learning. | All rights reserved. www.insightsias.com 60

It aims at promoting coordination between them by examining, discussing anddeliberating on such issues. Its duties, in detail, are as follows:

Investigating and discussing such subjects in which the states or the centrehave a common interest;making recommendations upon any such subject for the better coordinationof policy and action on it; andDeliberating upon such other matters of general interest to the states as maybe referred to it by the chairman.

The Council may meet at least thrice in a year. Its meetings are held in camera andall questions are decided by consensus.

Q Source:Chapter 15: Indian Polity: M Laxmikanth

91 The highest weightage in Consumer Price Index (Combined) is constituted byFuel and lightA.Clothing and associated itemsB.Food and associated itemsC.HousingD.

User Answer :Correct Answer : CAnswer Justification :

Learning: CPI is disaggregated at the rural and urban levels. The new overall allIndia CPI is a weighted average of the two.

Picture below: Left column CPI Rural, Middle Column CPI Urban, and Rightcolumn CPI Combined weights of various categories.

INSIG

HTS IAS

Page 61: Test 15 Solutions

TEST - 15

(C) Insights Active Learning. | All rights reserved. www.insightsias.com 61

You can read more about it here

http://www.onemint.com/2013/02/25/what-is-indias-cpi/

Q Source:Chapter 7: Indian Economy - Ramesh Singh

92 The Union Ministry of Environment, Forest and Climate Change (MoEFCC) hasreleased a new Four-colour Classification Scheme for industries based on their pollutionpotential. Which of these industries come under Red Category?

Paints1.Automobile Servicing2.Sugar industry3.Thermal Power Plants4.Select the correct answer using the codes below.

1 and 4 onlyA.1, 3 and 4 onlyB.2 and 4 onlyC.2 and 3 onlyD.

User Answer :Correct Answer : BAnswer Justification :

Learning:The four-colour classification (Red, Orange, Green, White) scheme ofindustrial sectors based on the Pollution Index (PI) which is a function of the (i)Emissions (air pollutants) (ii) Effluents (water pollutants) (iii) hazardous wastesgenerated and (iv) Consumption of resources.

Red category: PI score of 60 and above. These are severe polluting industries.Total 60 industries including sugar, thermal power plants, paints and othersare under in it.Orange category: PI score of 41 to 59. They moderately polluting industries.Total 83 industries like coal, washeries and automobile servicing are placedunder it.

Q Source:http://envfor.nic.in/legis/ucp/ucpsch8.html

93 The aim of Satya Sodhak Samaj was toLiberate the untouchable castes from exploitation and oppressionA.Promote the knowledge of the real SelfB.Liberate widows and poor women from social exploitationC.

INSIG

HTS IAS

Page 62: Test 15 Solutions

TEST - 15

(C) Insights Active Learning. | All rights reserved. www.insightsias.com 62

Promote the study of Vedas and ancient Indian cultureD.User Answer :Correct Answer : AAnswer Justification :

Learning:It is a society established by Jyotirao Phule in 1873. This was started as agroup whose main aim was to liberate the social shudra and untouchable castes fromexploitation and oppression.

Through his writings and activities Mahatma Phule condemned caste hierarchy andthe privileged status of priests in it.

He openly condemned the inequality in the religious books, orthodox nature ofreligion, exploitation of masses by the means of it, blind and misleading rituals, andhypocrisy in the prevalent religion.

Q Source:Chapter 6: Bipin Chandra: India’s struggle for Independence

94 In Producer Price Index the prices that are used for calculation excludesTaxes1.Profits2.Labour costs3.Land Rent4.Select the correct answer using the codes below.

3 and 4 onlyA.1 and 2 onlyB.2 onlyC.1, 2, 3 and 4D.

User Answer :Correct Answer : BAnswer Justification :

Learning:As the producers sell at higher prices to their wholesellers, so retailersand the price increase is translated into the higher consumer prices-thus the PPI isuseful in having an idea of the consumer prices in the future.

In PPI, only basic prices are used while taxes, trade margins and transport costs areexcluded. This index is considered a better measure of inflation as price changes atprimary and intermediate stages can be tracked before it gets built into the finishedgoods stage.

INSIG

HTS IAS

Page 63: Test 15 Solutions

TEST - 15

(C) Insights Active Learning. | All rights reserved. www.insightsias.com 63

Q Source:Chapter 7: Indian Economy - Ramesh Singh

95 Consider the following with reference to the Member of Parliament Local AreaDevelopment Scheme (MPLADS).

The projects are executed by District administration and the role of MPs is limited1.only upto recommendation of works.Nominated MPs are not covered under the scheme.2.Which of the above is/are correct?

1 onlyA.2 onlyB.Both 1 and 2C.NoneD.

User Answer :Correct Answer : AAnswer Justification :

Justification:Statement 1: Under the scheme, each MP has the choice to suggest tothe District Collector for, works to the tune of Rs. 5 Crore per annum to be taken upin his/her constituency.

It is the responsibility of the District Authority to sanction, execute and complete theworks recommended by Members of Parliament within the stipulated time period.

Statement 2: The Nominated Members of the Lok Sabha and Rajya Sabha mayselect any Districts from any State in the Country for implementation of their choiceof work under the scheme.

The Rajya Sabha MPs can recommend works in one or more districts in the Statefrom where he/she has been elected.

Q Source:MPLADS: Chapter 5: Indian Economy - Ramesh Singh

96 There was a general resentment against the Lex Loci Act that was proposed in 1845 andpassed in 1850. It was because

It provided for the confiscation of all property given to charitable andA.religious institutions.It provided the right to inherit ancestral property to Hindu converts toB.Christianity.Imparting religious education in local schools was declared illegal.C.The Act mandated reservations on religious lines in public institutions.D.

INSIG

HTS IAS

Page 64: Test 15 Solutions

TEST - 15

(C) Insights Active Learning. | All rights reserved. www.insightsias.com 64

User Answer :Correct Answer : BAnswer Justification :

Learning:The debate for a uniform civil code dates back to the colonial period inIndia.

The Lex Loci Report of October 1840 emphasised the importance and necessity ofuniformity in codification of Indian law, relating to crimes, evidences and contractbut it recommended that personal laws of Hindus and Muslims should be keptoutside such codification.

Q Source:Page 65: Chapter 6: Bipin Chandra: India’s struggle for Independence

97 Who among the following are in the Governing Council of the NITI Aayog?Union Planning Secretary1.Leader of Opposition of both houses2.Governor, RBI3.Chief Minister of all States of India4.Minister of Home Affairs5.Select the correct answer using the codes below.

1, 3 and 4 onlyA.2, 4 and 5 onlyB.4 and 5 onlyC.1, 2 and 3 onlyD.

User Answer :Correct Answer : CAnswer Justification :

Learning:Chairperson: Prime Minister of India; Ex officio Members, NITI Aayog;Vice-Chairperson, NITI Aayog; Full-time Members, NITI Aayog; Chief Ministersof all states and UTs; Administrator of all UTs are the members of the GoverningCouncil.

Ex-officio Members are Minister of Home Affairs; Minister of Finance; Minister ofCorporate Affairs; Minister of Information and Broadcasting; Minister of Railways;and Minister of Agriculture.

Full-Time Members are Shri Bibek Debroy; Shri V.K. Saraswat; and Prof. RameshChand.

INSIG

HTS IAS

Page 65: Test 15 Solutions

TEST - 15

(C) Insights Active Learning. | All rights reserved. www.insightsias.com 65

Q Source:Replaces Planning Commission: Chapter 5: Indian Economy - RameshSingh

98 Which of the following argument(s) were made by Dadabhai Naoroji about theeconomic aspects of the British rule in India?

The drain of wealth from India was the basic cause of India’s poverty.1.Foreign capital should not be preferred for India’s growth.2.Which of the above is/are correct?

1 onlyA.2 onlyB.Both 1 and 2C.NoneD.

User Answer :Correct Answer : CAnswer Justification :

Learning:He argued that it was the economic laws and policies of the British thatwere draining India and causing crisis in every sphere especially in loss oflivelihoods and loss of precious capital that could have been used for India’sdevelopment.

He and his contemporaries argued that using foreign capital for a nation’s growthwas undesirable and should be stopped. Building domestic capacity and funding itby domestic sources was the most desirable since no drain would be involved

Q Source:Page 73: Chapter 7: Bipin Chandra: India’s struggle for Independence

99 Hyperinflation can have which of the following consequence(s)?Loss of confidence in the domestic currency1.Increase in demand of assets other than money2.Which of the above is/are correct?

1 onlyA.2 onlyB.Both 1 and 2C.NoneD.

User Answer :Correct Answer : CAnswer Justification :

Justification:Statement 1 and 2: Inflation erodes the value of currency. For e.g. if

INSIG

HTS IAS

Page 66: Test 15 Solutions

TEST - 15

(C) Insights Active Learning. | All rights reserved. www.insightsias.com 66

inflation in India is 5000%, then the very next year the value of Rs. 5000 in year2015 will be near Rs. 100 in year 2016. It also leads to depreciation in the exchangerate of the currency.

As a result, people lose confidence in the currency and holding it becomes a riskyproposition. So, people switch to other forms of wealth like Gold, Foreign Currency(also known as “inflation proof” assets) etc.

It is one of the important reasons for the recent Gold rush in India. Introducinginflation-indexed bonds, Gold bonds is one way of reducing the demand ofunproductive physical assets.

Q Source: Chapter 7: Indian Economy - Ramesh Singh

100 Article 355 of the Constitution of India is an important instrument that enables theCentre to exercise control over State administration. It imposes which of the followingduties on the Centre?

To protect every state against external aggression and internal disturbance1.To protect the financial integrity of the states2.To ensure that the government of every state is carried on in accordance with the3.provisions of the ConstitutionTo resolve conflicts between States through negotiations or binding executive orders4.Select the correct answer using the codes below.

1 and 3 onlyA.2, 3 and 4 onlyB.4 onlyC.1, 3 and 4 onlyD.

User Answer :Correct Answer : AAnswer Justification :

Learning:There is an old The Hindu Article that should be read for understandingthe Article 355 better.

http://www.thehindu.com/2002/05/04/stories/2002050401351200.htm

Financial emergency under article 360 of the constitution deals with the 2ndstatement.

Q Source:Chapter 14: Indian Polity: M Laxmikanth

INSIG

HTS IAS